LSAT and Law School Admissions Forum

Get expert LSAT preparation and law school admissions advice from PowerScore Test Preparation.

 Administrator
PowerScore Staff
  • PowerScore Staff
  • Posts: 8923
  • Joined: Feb 02, 2011
|
#35404
Complete Question Explanation

Parallel Flaw. The correct answer choice is (D)

The stimulus features conditional reasoning. Since no one who works at Leila’s has received both a poor performance evaluation and a raise, the conditional relationship in the first sentence can be diagrammed as follows:
  • Poor Evaluation ..... :arrow: ..... Raise

    Raise ..... :arrow: ..... Poor Evaluation
The abbreviated form of this relationship (using the Double-Not Arrow) takes into account both the original statement and its contrapositive:
  • Poor Evaluation ..... :dblline: ..... Raise
The first clause of the second sentence indicates that one of these two conditions has not occurred, as Lester did not receive a raise:
  • RaiseLester
On the basis of this information, the author concludes that the other condition must have occurred, i.e. that Lester must have received a poor performance evaluation:
  • Conclusion: ..... RaiseLester ..... :arrow: ..... Poor Evaluation
This conclusion is fundamentally flawed. The fact that two events cannot occur at the same time does not mean that one of them has to occur: it is entirely possible that Lester received neither a poor performance evaluation nor a raise. Technically speaking, the conclusion takes the form of a Mistaken Negation: A and B cannot occur simultaneously, so if A does not occur, B must occur. (The correct conclusion should have stated that if A occurs, B cannot occur, and vise versa). The correct answer choice must contain the same logical flaw.

Answer choice (A): This answer choice is incorrect because the argument in it is valid:
  • Premise: ..... Own ..... :dblline: ..... Pay Rent

    Conclusion: ..... Pay RentNeighbors ..... :arrow: ..... OwnNeighbors
Answer choice (B): This answer choice is virtually identical to incorrect answer choice (A):
  • Premise: ..... Own ..... :dblline: ..... Pay Rent

    Conclusion: ..... OwnNeighbors ..... :arrow: ..... Pay RentNeighbors
Answer choice (C): This answer choice can be immediately eliminated because it fails two of the tests in our Parallel Reasoning Elemental Attack. First, we have no evidence of two separate events being mutually exclusive (i.e. the premises do not match); second, the conclusion is logically valid (i.e. it fails the Validity Test).
  • Premise: ..... Rent ..... :arrow: ..... Own

    Conclusion: ..... RentNeighbors ..... :arrow: ..... OwnNeighbors
Answer choice (D): This is the correct answer choice, as the argument makes an identical mistake in reasoning. One cannot simultaneously own and rent a house. So, if the neighbors do not own their house, they must be paying rent on it:
  • Premise: ..... Own ..... :dblline: ..... Pay Rent

    Conclusion: ..... OwnNeighbors ..... :arrow: ..... Pay RentNeighbors
You should not be concerned with the fact that the order of the two premises is reversed. Their order of presentation is irrelevant as long as the pattern of reasoning stays the same.

Answer choice (E): This answer choice is virtually identical to incorrect answer choice (C):
  • Premise: ..... Own ..... :arrow: ..... Rent

    Conclusion: ..... OwnNeighbors ..... :arrow: ..... RentNeighbors
Since this is a valid argument with no evidence of two mutually exclusive conditions, answer choice (E) is incorrect.
 Arindom
  • Posts: 76
  • Joined: Apr 11, 2016
|
#23583
Hi,

So, I think I diagrammed the stimulus incorrectly. Can you please check where I went wrong?
WL - Works at Leila's
PP - Poor Performance
R - Raise
Lester

Principle: WL ---- Not PP and Not Raise
Not Raise (lester)
----------------------------------
C: PP (Lester must have received PP)

I picked ans choice A on this basis, which i diagrammed as -
LH - Lives in a house
R- Rent
O - Owns it

Principle: LH ---Not O and Not R
R (neighbor's pay rent)
-------------------------------
C: Not O (neighbors do not own it)

Thanks,

- Arindom
 Robert Carroll
PowerScore Staff
  • PowerScore Staff
  • Posts: 1787
  • Joined: Dec 06, 2013
|
#23595
Arindom,

You have to be careful, because the stimulus says that no one who works at Leila's has both of the qualities mentioned, not that anyone who works there lacks both qualities. In other words, the negation of "both A and B" is "not A or not B".

Here, it is as follows:

WL :arrow: PP or R

R(Lester)

Therefore, PP(Lester).

The stimulus is claiming that Lester has one of the necessary conditions, and that he therefore has the negation of the other necessary condition. This is not true.

Answer choice (A) is as follows:

LH :arrow: O or R

R(neighbors)

Therefore, O(neighbors)

Answer choice (A) is claiming that the neighbors have the negation of one of the necessary conditions, and that they therefore have the other necessary condition. This is not the same pattern as the stimulus.

Robert Carroll
 Arindom
  • Posts: 76
  • Joined: Apr 11, 2016
|
#23599
Ahh, yes. I get it.

Thank you!
 Oneshot06
  • Posts: 21
  • Joined: Feb 07, 2018
|
#44216
Administrator wrote:Complete Question Explanation

Parallel Flaw. The correct answer choice is (D)

The stimulus features conditional reasoning. Since no one who works at Leila’s has received both a poor performance evaluation and a raise, the conditional relationship in the first sentence can be diagrammed as follows:
  • Poor Evaluation ..... :arrow: ..... Raise

    Raise ..... :arrow: ..... Poor Evaluation
The abbreviated form of this relationship (using the Double-Not Arrow) takes into account both the original statement and its contrapositive:
  • Poor Evaluation ..... :dblline: ..... Raise
The first clause of the second sentence indicates that one of these two conditions has not occurred, as Lester did not receive a raise:
  • RaiseLester
On the basis of this information, the author concludes that the other condition must have occurred, i.e. that Lester must have received a poor performance evaluation:
  • Conclusion: ..... RaiseLester ..... :arrow: ..... Poor Evaluation
This conclusion is fundamentally flawed. The fact that two events cannot occur at the same time does not mean that one of them has to occur: it is entirely possible that Lester received neither a poor performance evaluation nor a raise. Technically speaking, the conclusion takes the form of a Mistaken Negation: A and B cannot occur simultaneously, so if A does not occur, B must occur. (The correct conclusion should have stated that if A occurs, B cannot occur, and vise versa). The correct answer choice must contain the same logical flaw.

Answer choice (A): This answer choice is incorrect because the argument in it is valid:
  • Premise: ..... Own ..... :dblline: ..... Pay Rent

    Conclusion: ..... Pay RentNeighbors ..... :arrow: ..... OwnNeighbors
Answer choice (B): This answer choice is virtually identical to incorrect answer choice (A):
  • Premise: ..... Own ..... :dblline: ..... Pay Rent

    Conclusion: ..... OwnNeighbors ..... :arrow: ..... Pay RentNeighbors
Answer choice (C): This answer choice can be immediately eliminated because it fails two of the tests in our Parallel Reasoning Elemental Attack. First, we have no evidence of two separate events being mutually exclusive (i.e. the premises do not match); second, the conclusion is logically valid (i.e. it fails the Validity Test).
  • Premise: ..... Rent ..... :arrow: ..... Own

    Conclusion: ..... RentNeighbors ..... :arrow: ..... OwnNeighbors
Answer choice (D): This is the correct answer choice, as the argument makes an identical mistake in reasoning. One cannot simultaneously own and rent a house. So, if the neighbors do not own their house, they must be paying rent on it:
  • Premise: ..... Own ..... :dblline: ..... Pay Rent

    Conclusion: ..... OwnNeighbors ..... :arrow: ..... Pay RentNeighbors
You should not be concerned with the fact that the order of the two premises is reversed. Their order of presentation is irrelevant as long as the pattern of reasoning stays the same.

Answer choice (E): This answer choice is virtually identical to incorrect answer choice (C):
  • Premise: ..... Ownr ..... :arrow: ..... Rent

    Conclusion: ..... OwnNeighbors ..... :arrow: ..... RentNeighbors
Since this is a valid argument with no evidence of two mutually exclusive conditions, answer choice (E) is incorrect.
Hi, I just want to nail the conditional relationship in this question as this sort of conditional reasoning comes up often, especially in logic games.

The part about if PPE :arrow: raise and if Raise :arrow: PPE confuses me a little. I understand that PPE :dblline: raise, because they both cannot occur at same time (although they both don't have to occur, hence why the conclusion is flawed). I'm just a little confused by the notation of if PPE :arrow: raise and vice versa because it tells me that if one occurs, the other MUST occur, not that they both cannot occur at same time.

Thanks!
 Malila Robinson
PowerScore Staff
  • PowerScore Staff
  • Posts: 296
  • Joined: Feb 01, 2018
|
#44259
Hi One,
It appears that there is a missing negation in there. It should be PPE ---> no raise, and the Contra Positive is Raise ---> no PPE, this is what give you the PPE <-/-> Raise. So while you cannot have both together it doesn't say that you can't have a situation where neither of them occur (no raise and no PPE). That is the same mistaken reasoning that is in (D)

Malila
User avatar
 Jonathan Evans
PowerScore Staff
  • PowerScore Staff
  • Posts: 726
  • Joined: Jun 09, 2016
|
#44280
Thanks, Malila and OneShot! The explanation has been fixed.
 ericau02
  • Posts: 73
  • Joined: Feb 19, 2019
|
#64729
Hi can someone please please please explain this I am having trouble seeing the difference in these ac's. for c and d arent they exactly the same diagramming them its showing not one than the other im so confused.
 Brook Miscoski
PowerScore Staff
  • PowerScore Staff
  • Posts: 418
  • Joined: Sep 13, 2018
|
#64988
erica,

Answer choice (C):
Live and -Rent :arrow: Own
Neighbors don't rent :arrow: Own

This logic is good.

Answer choice (D):

Own :dblline: Rent
Neighbors -Own :arrow: Rent

This logic is a Mistaken Negation.

The stimulus states:

Poor Eval :dblline: Raise
Lester -Raise :arrow: Poor Eval

This logic is a Mistaken Negation; thus, the answer is (D).
User avatar
 pmuffley
  • Posts: 39
  • Joined: Sep 24, 2021
|
#92357
Trying not to build up bad habits here so I want to know if this is an effective way to approach this problem. I am trying to improve my timing. Diagramming everything out took me forever. Doing it this way took me 2 minutes.

My (almost literal) thought process was as follows:

1. Stim says two things never happen together.
2. Stim says one thing not happening causes another to happen

abstract: lightening and sun never happen together. Therefore, if it is not lightening, then it is sunny. That's wrong because it could be cloudy and not lightening.

3. Answer will show:
a. two things can't happen together
b. one not happening
c. as a result, the other happens

a. wrong. result says something didn't happen not that it happened
b. wrong. Again, result says something didn't happen not that it happened
c. it is saying either/or you rent or you own. It actually makes sense.
d. looks good
e. same thing as c.

Is this going to get me in trouble in other problems?

Get the most out of your LSAT Prep Plus subscription.

Analyze and track your performance with our Testing and Analytics Package.